Solving a system with the inverse of a matrix.

Click For Summary
The discussion revolves around solving a system of equations using the inverse of a matrix. The user successfully found the inverse of matrix A but is confused about how to apply it to the system of equations presented in part b. They initially struggle to see the connection between the two matrices and suspect an error in the problem setup. After consulting with a teaching assistant, they confirm that the equations should indeed relate to the inverse matrix. The clarification allows them to proceed with the solution.
thatguythere
Messages
91
Reaction score
0

Homework Statement


a)Use Gauss-Jordan elimination to find the inverse of A =
[ 2 1 4 ]
[ 1 1 2 ]
[ -2 -3 -2 ]

b) Use the result from part a) to find the solution of the following system.

5x+2y-3z = 5
x+y-z = -1
-3x-y+2z = 2

Homework Equations





The Attempt at a Solution



My problem is not with part a), I quite easily found the inverse of the matrix. What I am not understanding is what exactly they are asking me to do for part b). How can I use the inverse of one matrix to solve for another? Any help is greatly appreciated.
 
Physics news on Phys.org
If you change

5x+2y-3z = 5
x+y-z = -1
-3x-y+2z = 2

into a matrix equation in the form of Bx=C, what would B, X and C be?
 
rock.freak667 said:
If you change

5x+2y-3z = 5
x+y-z = -1
-3x-y+2z = 2

into a matrix equation in the form of Bx=C, what would B, X and C be?

I'm with thatguythere. I can't see that the matrix in part b) is related in any simple way to the matrix in part a). I'm suspecting the somebody goofed when assembling the problem.
 
I contacted my TA and it is indeed a mistake. It should be
2x + y + 4z = 5
x + y + 2z = -1
-2x -3y -2z = 2
I should be able to manage now, thanks.
 
Dick said:
I'm with thatguythere. I can't see that the matrix in part b) is related in any simple way to the matrix in part a). I'm suspecting the somebody goofed when assembling the problem.

Ah well, I didn't calculate the inverse, so I assumed it one of those problems where the matrix equation would be A-1x=B
 
Question: A clock's minute hand has length 4 and its hour hand has length 3. What is the distance between the tips at the moment when it is increasing most rapidly?(Putnam Exam Question) Answer: Making assumption that both the hands moves at constant angular velocities, the answer is ## \sqrt{7} .## But don't you think this assumption is somewhat doubtful and wrong?

Similar threads

  • · Replies 2 ·
Replies
2
Views
933
  • · Replies 3 ·
Replies
3
Views
2K
  • · Replies 6 ·
Replies
6
Views
2K
  • · Replies 12 ·
Replies
12
Views
3K
Replies
8
Views
3K
  • · Replies 4 ·
Replies
4
Views
1K
Replies
3
Views
2K
  • · Replies 9 ·
Replies
9
Views
2K
  • · Replies 2 ·
Replies
2
Views
1K
  • · Replies 3 ·
Replies
3
Views
1K